O&G #1 Flashcards

1
Q

A 26-year-old multigravid woman is in spontaneous labour at 41 weeks. She
has no antenatal risk factors with normal ultrasound scans. On examination,
the head is 2/5th palpable per abdomen. She has a spontaneous
rupture of membranes at 3 cm with heavily blood-stained liquor. The
CTG shows significant abnormalities. The midwife performs a vaginal
examination and there is no cord protruding through the cervix which is
now 4 cm dilated. The mother feels no pain.
What is the most likely diagnosis?

A. Bloody show

B. Placental abruption

C. Placenta praevia

D. Uterine rupture

E. Vasa praevia

A

E – Vasa praevia

Vasa praevia is rare, occurring in only 1 in 3000 pregnancies. The umbilical cord
vessels travel away from the placenta in the membranes and overlie the internal
cervical os. The vessels can tear leading to rapid exsanguination of the fetal
circulation. The risk of vessels tearing is greatest when cervical dilation occurs
and at rupture of membranes. A severely abnormal cardiotocograph (CTG)
is seen with a small amount (,500 mL) of painless vaginal blood loss.
Because it is fetal blood that is lost in vasa praevia, fetal mortality is very high
(35–95%) while there is little risk to the mother. A Caesarean section must be
performed immediately and the neonate transfused. There is no specific
investigation for vasa praevia, so the diagnosis is clinical and only confirmed
when the placenta and membranes are examined after Caesarean section.
Vasa, plural of Latin vas ¼ vessel.

How well did you know this?
1
Not at all
2
3
4
5
Perfectly
2
Q

Which of the following conditions is an indication for routine delivery by
Caesarean section?

A. Hepatitis C virus

B. Maternal request

C. Preterm birth

D. Previous Caesarean section

E. Twin pregnancy with first twin breech and second twin cephalic

A

E – Twin pregnancy with first twin breech and second twin cephalic

A Caesarean section should be routinely offered to the following women:
† HIV with or without other concurrent infections
† Primary genital herpes in the third trimester (NB not a secondary attack)
† Placenta praevia major, i.e. grade 3 or 4
† Twin pregnancy where the first baby is breech
† Singleton breech at term but only after external cephalic version has been
offered and failed or contraindicated
These women should not routinely be offered a Caesarean section:
† Twin pregnancy where the first twin is cephalic
† Preterm birth
† Small for gestational age baby
† Hepatitis B virus without HIV
† Hepatitis C virus without HIV
† Recurrent genital herpes at term
Maternal request is not an indication for a Caesarean section and the obstetrician
must discuss in full why the woman wants to have a Caesarean and her
concerns about normal labour. She must also be informed about the risks and
benefits of Caesarean section compared with normal labour. Counselling can
be offered if the woman has a fear of childbirth. The obstetrician may refuse
to perform a section after a maternal request but is obligated to refer the
woman to another clinician for a second opinion.

How well did you know this?
1
Not at all
2
3
4
5
Perfectly
3
Q

A 25-year-old woman attends the GUM clinic complaining of increased
vaginal discharge which has an unpleasant odour. She says sexual intercourse
with her partner is uncomfortable. A swab is taken and sent to the
lab. On direct microscopy a flagellated protozoan is seen.
Which is the most likely pathogen?

A. Candida albicans

B. Chlamydia trachomatis

C. Gardnerella vaginalis

D. Neisseria gonorrhoeae

E. Trichomonas vaginalis

A

E – Trichomonas vaginalis

Trichomoniasis is a sexually transmitted infection caused by the flagellated
protozoan Trichomonas vaginalis, which invades superficial epithelial cells of
the vagina, urethra, glans penis, prostate and seminal vesicles. Affected
females present with an offensive frothy greeny-grey discharge, vulval soreness,
dyspareunia, dysuria, vaginitis and vulvitis, although some are asymptomatic.
On examination, the cervix may have a punctate erythematous (strawberry)
appearance. Males are mostly asymptomatic. Diagnosis is by direct microscopy
or culture of vaginal exudate. Treatment is with metronidazole.
Gonorrhoea is caused by the Gram-negative diplococcus Neisseria gonorrhoeae
infecting the mucosal surfaces of the genitourinary tract, rectum and
pharynx. The majority of females are asymptomatic (70%). Around 85–90%
of cases involve the cervix but only 10% of these cases have a significant increase
in vaginal discharge. Seventy percent of cases involve the urethra and again it is
generally asymptomatic although dysuria and urinary frequency may be seen.
The vagina is not infected. Complications include Bartholin’s abscess and gonococcal
salpingitis with irreversible tube damage. Infected males present with
dysuria, frequency and/or a mucopurulent discharge after 3 to 5 days,
coupled with urethritis and meatal oedema. Disseminated gonococcal infection
occurs in ,1% cases and causes pyrexia, a vasculitic rash and polyarthritis.
Culture sensitive antibiotics are used for treatment.
Chlamydia is caused by the oculogenital serovars D–K of Chlamydia trachomatis.
Infection tends to be asymptomatic, although there can be increased vaginal
discharge (30%), dysuria and urinary frequency. On examination, a ‘cobblestone’
appearance of the cervix may be noted. Ascending infection can cause
salpingitis and, if it enters the abdominal cavity, perihepatitis (Fitz-Hugh–Curtis
syndrome) which leads to right upper quadrant pain and tenderness. Chlamydia
is a major cause for infertility and increases the possibility of ectopic pregnancy. In
males, symptoms include mucopurulent discharge and dysuria (asymptomatic in
25%). Epididymo-orchitis is a complication. Diagnosis of chlamydia infection is by
urine antigen detection or vaginal swab culture. Treatment is with doxycycline.

How well did you know this?
1
Not at all
2
3
4
5
Perfectly
4
Q

A 57-year-old woman presents with a history of having to run to the toilet
and occasionally not getting there in time. She needs to wear pads every day
and this is negatively impacting on her life. She also complains of waking
up two or three times per night to pass urine. She has had two children
by normal delivery and has never had any surgery on her bladder. She
says she has been doing occasional pelvic floor exercises with little success.
Considering her diagnosis, what is the first-line treatment?

A. Bladder training

B. Botulinum toxin

C. Oxybutynin

D. Pelvic floor exercises with a trained physiotherapist

E. Tolteridone

A

A – Bladder training

Urinary incontinence affects 10–20% of the adult female population. It is a
social and hygienic problem. Incontinence occurs when the intravesical pressure
exceeds the urethral closure pressure.
This lady is suffering from an overactive bladder (OAB), also known as detrusor
instability, unstable bladder or hyperactive bladder. First-line treatment is
bladder training for 6 weeks.
OAB is the second most common type of urinary incontinence in females (after
genuine stress in continence). Other causes include retention with overflow,
fistula and congenital abnormalities. OAB is where an involuntary detrusor
contraction results in leakage of urine. It can occur in conjunction with
genuine stress incontinence. Women complain of frequency of micturition
and urgency, which they describe as an overwhelming desire to pass urine
with associated urge incontinence. This can also be associated with nocturnal
enuresis. It is important to establish how it affects her life, dependent on severity,
lifestyle and occupation, as this will impact on how aggressive the treatment
strategies will be.

How well did you know this?
1
Not at all
2
3
4
5
Perfectly
5
Q

You are looking at a CTG of a woman of 39 weeks gestation who has come to
the antenatal day unit as she has had reduced fetal movements. There is a
baseline rate of 170. There are four accelerations in a 20-minute section.
The variability is over 10 beats. There are no decelerations.
What could explain the features of this trace?

A. Maternal pyrexia

B. Normal trace

C. Pre-terminal trace

D. Sleep pattern of fetus

E. Thumb sucking of fetus

A

A – Maternal pyrexia

Cardiotocography (CTG) measures the fetal heart rate with uterine activity. It
can be used confidently after 32 weeks gestation to monitor the condition of
a fetus in correlation with the clinical situation. Prior to this gestation, the autonomic
nervous system is not sufficiently developed to produce the predictable
responses of the more mature fetus. A normal CTG is reassuring, but an
abnormal CTG is not always pathological.
Indications for CTG monitoring can be maternal (previous Caesarean section,
pre-eclampsia, diabetes, antepartum haemorrhage), fetal (intrauterine growth
restriction, prematurity, oligohydramnios, multiple pregnancy, breech) or intrapartum
(oxytocin use, epidural use, induction of labour). The CTG lead is placed
on the mother’s abdomen or attached vaginally to the fetal scalp.
The use of CTG does not appear to improve long-term neonatal outcome.

How well did you know this?
1
Not at all
2
3
4
5
Perfectly
6
Q

You are examining a woman in established labour with the midwife and she
asks you to tell her how you would describe the examination. The cervix is
fully dilated. Anteriorly you feel a diamond-shaped fontanelle and if you
follow a line posteriorly you can then feel a Y-shaped depression in the
skull bones.
How is the position best described?

A. Brow

B. Left occipitotransverse

C. Occipitoanterior

D. Occipitoposterior

E. Right occipitotransverse
Practice

A

D – Occipitoposterior

Vaginal examination often involves a speculum examination and a digital
examination, but in this case there is no need for a speculum as it is already
known that she is in established labour.

During labour, digital examinations are used to assess dilation of the cervix to
ensure adequate progression. This is described in centimetres of dilation and
how effaced the cervix is. The fetal head is also examined to assess its position
using fontanelles. Fontanelles are the soft spots on the baby’s head in
between the skull bones. They are covered with fibrous tissue and allow the
bones to overlap to allow the head to pass more easily through the birth
canal. The anterior fontanelle is a diamond-shaped structure formed at the
junction of the two frontal bones, which cover the forehead, and the two
parietal bones which cover the side of the head. This fontanelle will become
ossified by the child’s second birthday. The posterior fontanelle is Y-shape and
is made from the junction between the two parietal bones and the occipital
bone. This fontanelle will close during the first few months of life. The line felt
in the question was the sagittal suture which runs from the anterior to the
posterior fontanelle between the two parietal bones.

How well did you know this?
1
Not at all
2
3
4
5
Perfectly
7
Q

In which anatomical location does fertilization normally occur?

A. Ampulla of fallopian tube

B. Cervix

C. Fimbriae of fallopian tube

D. Infundibulum of fallopian tube

E. Uterus

A

A – Ampulla of fallopian tube

Fertilization is defined as the union of the ovum and the spermatozoon. The
ovum is normally released from the ovarian follicle every 28 days. The fimbrial
end of the fallopian tube lies close to the release spot so the ovum is taken up
into the fallopian tube. The lumen of the fallopian tube is lined by cilia and it
is the combination of the rhythmical movement of these cilia and the peristalsis
created by the muscles of the fallopian tube that moves the ovum along towards
the uterus. The ovum has a physiological pause of up to 38 hours when it
reaches the ampulla of the fallopian tube. It is this physiological pause that
means that fertilization normally happens in the ampulla. It also allows extra
time for the sperm and ovum to be in the ‘same place at the same time’ as
sexual intercourse happens at random in humans. The sperm can survive up
to 48 hours, so as long as sexual intercourse occurred within 2 days of ovulation,
fertilization is feasible. After the pause, and possible fertilization, the rhythmic
waves and peristalsis recommence and the ovum moves down into the uterus.
The lateral funnel-shaped end of the fallopian tube is called the infundibulum
which opens into the peritoneal cavity through the abdominal ostium. There
are many finger-like projections known as fimbriae at the end of the infundibulum
which spread over the medial surface of the ovary to waft the released ovum
into the fallopian tube. The ampulla is medial to the infundibulum and is the
longest and widest part: this is where fertilization occurs. The isthmus enters
the uterine horn and is thick-walled. The uterine portion travels through the
uterine wall to terminate in the uterine cavity at the uterine ostium.

How well did you know this?
1
Not at all
2
3
4
5
Perfectly
8
Q

A 21-year-old lady at 40 weeks þ 12 days is being induced. She has received
two doses of prostaglandins after examination revealed a low Bishop
score. She is experiencing mild contractions with good fetal movements.
Her CTG trace is reactive. She is fed up, tired and is becoming angry
with the midwives as she thought she would have delivered sooner.
On abdominal palpation there is cephalic presentation with two-fifths
palpable. On vaginal examination after the second dose of progesterone
she is 3 cm dilated with a partially effaced cervix.
What would be the next course of action?

A. Artificial rupture of membranes

B. Caesarean section

C. Further prostaglandin

D. Observation alone

E. Oxytocin

A

A – Artificial rupture of membranes

Induction of labour is offered if pregnancy continues past 40 weeks þ12 days.
The process involves vaginal prostaglandins with artificial rupture of membranes
(ARM) and use of oxytocin.
This lady has received two doses of vaginal prostaglandins (PGE2) to initiate
contractions and encourage cervical ripening. The tablets are 3 mg and are
given 6–8 hourly with a maximum dose of 6 mg/day. They have clearly
worked as she has progressed from a low Bishop score to a cervical dilation of
3 cm. If women are progressing well with strong contractions no other action
is needed, however if there is slow progress with minimal contractions (in this
case) an ARM can be performed and an oxytocin infusion is used to maintain
the contractions after membrane rupture. Further prostaglandins are contraindicated
due to the risk of hyperstimulation as she already feels some uterine activity.
An amnihook is used to pierce the membranes surrounding the baby. The
colour of the amniotic fluid should be recorded and any meconium or blood
should be noted and appropriate action should be taken if required. The fetal
heart should always be checked after an ARM. If there is a high head an ARM
may be performed in theatre due to the higher risk of a cord prolapse where
an immediate caesarean section may be needed.
Complications of induction and augmentation of labour include:
† Failure of induction – requiring operative delivery
† Uterine hyperstimulation (.7 contractions/15 minutes) – this can cause
maternal and fetal distress. An oxytocin infusion must be stopped and continuous
monitoring is needed. Tocolysis (suppression of contractions) can
be used but, if there is suspicion of fetal compromise, delivery is needed as
soon as possible.
† Nausea, vomiting, diarrhoea – systemic side-effects of prostaglandins
† Water intoxication – as oxytocin is infused with fluid
† Uterine rupture

How well did you know this?
1
Not at all
2
3
4
5
Perfectly
9
Q

Which measurement is the most reliable indicator of gestational age in the
first trimester?

A. Biophysical profile

B. Biparietal diameter

C. Crown–rump length

D. Femur length

E. Nuchal translucency

A

C – Crown–rump length

Ultrasound scanning is a means of monitoring pregnancy and to date is without
proven maternal or fetal risk. Scans can be performed abdominally or transvaginally.
The scan at 10–14 weeks is used to date the fetus by measuring crown–rump
length (CRL). The estimated delivery date (EDD) is calculated from the last menstrual
period (LMP) unless the ultrasound scan (USS) date differs by more than
one week. If there is a discrepancy of more than one week between the date
calculated from LMP and USS then it should be amended to be the EDD from
the CRL. The biparietal diameter or head circumference is used over 14 weeks
as the fetus becomes more flexed in shape so CRL is less accurate.
EDD can be calculated from the LMP however LMP may be not be accurately
recalled, there may be an irregular cycle and bleeding early in the pregnancy
may be mistaken as a period. The EDD is 40 weeks after the first day of the
LMP and not from the date of conception however this is only true if the
cycle is 28 days and regular. If the cycle is known to be shorter or longer then
days can be added or subtracted accordingly. The first date of a positive pregnancy
test can be useful as it will become positive on the day the next period
would have been due so when a positive pregnancy test is seen the gestation
must be at least 4 weeks.
Every woman should be offered an ultrasound scan early in pregnancy between
10–14 weeks. There are a number of reasons for this:
† To establish viability and ensure that either a molar pregnancy or missed
miscarriage has not occurred
† To detect multiple pregnancies and to determine chorionicity and amnionicity
(most reliably done in the first trimester)
† The nuchal translucency test is done between 11–14 weeks to assess risk
for Down’s syndrome
† Gross anatomical anomalies can be detected in the first trimester scan,
such as major anterior abdominal wall defects, cystic hygroma, anencephaly
and bladder outflow obstruction

How well did you know this?
1
Not at all
2
3
4
5
Perfectly
10
Q

A 32-year-old lady returns to the gynaecology clinic to find out the
results of her cervical screening test. You see her report says moderate
dyskaryosis.
What would be the next stage in her management?

A. Colposcopy

B. Recall in 6 months

C. Recall in 1 year

D. Recall in 3 years

E. Repeat the test today

A

A – Colposcopy

If a woman aged 25 to 49 years has a normal smear they are called back in
3 years. A woman may be recalled if there are inadequate cells for the study –
this would normally be in 6 months. If a diagnosis of mild dyskaryosis is made
a repeat is needed in 6 months as these cells often revert to normal without
any treatment. If on the repeat test at 6 months the cells still show mild dyskaryosis,
colposcopy will be required. A single diagnosis of moderate or severe
dyskaryosis indicates referral to colposcopy. Obviously a diagnosis of invasive
carcinoma would require immediate specialist referral. Immunocompromised
patients require annual screening

How well did you know this?
1
Not at all
2
3
4
5
Perfectly
11
Q

Which one of the following factors increases your risk of developing
ovarian cancer?

A. Early menopause

B. Late menarche

C. Multiparity

D. Nulliparity

E. Oral contraceptive pill

A

D – Nulliparity

Ovarian cancer is the most common gynaecological cancer in the UK and the
fifth most common cancer in the UK overall, and the incidence is rising. It is
seen mainly in the fifth, sixth and seventh decade. It has a poor 5-year survival
rate of only 25%. There is no premalignant phase so no screening test can be
developed.
Like all cancers there are numerous risk factors for its development. It may be
related to ovulation, due to the repair of the ovarian epithelium required following
each ovulation. This means the more you ovulate the more you increase your
risk of developing cancer of the ovary. Hence nulliparity, infertility, late menopause
and early menarche all increase your risk, whereas risk is lowered by the
contraceptive pill, breastfeeding and pregnancy. Pelvic surgery decreases the
risk (including hysterectomy, unilateral oophorectomy and sterilization) for
reasons that are not fully understood.
The risk of ovarian cancer is slightly increased with a positive family history and
this much more significant if there was early onset and more than one primary
relative affected. Around 5 to 10% of ovarian cancers have a direct genetic link
with the most significant being BRCA1 and BRCA2. Affected women have a lifetime
risk of up to 50% of developing ovarian cancer, hence close monitoring is
needed using CA125 and pelvic ultrasounds. Furthermore a prophylactic
bilateral oophorectomy may be considered by some once the family is complete.
Other cancers that are linked are breast, endometrial and colonic.

How well did you know this?
1
Not at all
2
3
4
5
Perfectly
12
Q

A 22-year-old woman attends the labour ward for induction of labour as she
is 40 weeks þ 12 days. She has had an uncomplicated pregnancy. She has no
pain in her abdomen and says that the baby is moving but less than normal.
A CTG is performed and the baseline is 135, variability is over 10, accelerations
are present and there are no decelerations. You examine her and
find a cephalic presentation and a long and closed cervix.
What would you like to do next?

A. Artificial rupture of membranes

B. Elective Caesarean section

C. Emergency Caesarean section

D. Oxytocin

E. Prostaglandin
Practice

A

E – Prostaglandin

Induction of labour (IOL) is used as there is increased risk of stillbirths with
increasing gestation over 37 weeks. The current data suggest the stillbirth rate is 1 per 3000 at 37 weeks, 3 per 3000 at 42 weeks and 6 per 3000 at 43 weeks.
About 20% women need IOL. If a woman declines IOL at 42 weeks they should
be offered twice-weekly cardiotocograph (CTG) and ultrasound scans to
measure maximum amniotic pool depth.
A stretch and sweep should be done prior to formal IOL. A finger is inserted into
the cervix and the membranes are stripped from the uterine wall with the aim of
inducing labour more naturally.
Prior to IOL the woman’s cervix should be assessed using the Bishop score. If the
Bishop score is very low, like this case, IOL involves vaginal prostaglandins (PGE2)
as either tablets or gels to initiate contractions and encourage cervical ripening.
The tablets are 3 mg and are given 6 to 8 hourly with a maximum dose of
6 mg. The CTG has to be reassuring for prostaglandins to be given and there
should be no pain or evidence of contractions otherwise you increase the risk
of uterine hyperstimulation.
If women are progressing well they can be left to labour; however, if there is slow
progress or the cervix is already dilated on initial examination an artificial rupture
of membranes can be performed along with an oxytocin infusion to maintain the
contractions. If there has been pre-labour rupture of membranes, the oxytocin
can be started regardless of the state of the cervix.

How well did you know this?
1
Not at all
2
3
4
5
Perfectly
13
Q

A 27-year-old woman is of 19 weeks gestation. She has a 3-day history of
flu-like symptoms with a macular rash over her body. Her doctor takes
serological testing. When he has the results he tells her that her baby is
at increased risk of sensorineural deafness, cataracts, congenital heart
disease, learning difficulties, hepatosplenomegaly and microcephaly.
What is the underlying causative agent?

A. Chickenpox

B. Cytomegalovirus

C. Listeria

D. Parvovirus

E. Rubella

A

E – Rubella

Rubella (German measles) is a viral infection spread by person-to-person
contact. It has an incubation period of 14 to 21 days. Rubella is rare nowadays
thanks to immunization (MMR vaccine), and the UK immunity is 97%. Women
develop a non-specific flu-like illness with a macular rash covering their trunk (20
to 50% infections are asymptomatic). Diagnosis is confirmed by serological antibody
testing. Rubella antibodies are checked at booking and postnatal vaccination
is offered to those with low titres. There is an 80% risk of infection to
the fetus if rubella develops in the first trimester, dropping to 25% at the end of the second trimester. Teratogenic effects are worse at earlier gestations, with
a 50% risk of abnormalities if the fetus is under 4 weeks, 25% at 5–8 weeks, 10%
at 9–12 weeks and 1% over 13 weeks. The characteristic abnormalities from
maternal rubella infection are sensorineural deafness, cataracts, congenital
heart disease, learning difficulties, hepatosplenomegaly, jaundice, microcephaly
and spontaneous miscarriage.
Rubella is also known as German measles as the first three reported cases were
described by German physicians who all thought the condition was a variant of
measles.
Listeria is a rare bacterial infection. The Gram-positive coccus, Listeria monocytogenes,
is found in soil, animal faeces, paˆte´ and unpasteurized dairy products
such as soft cheese. Pasteurized milk carries no risk. The incubation period is 3
to 70 days. The incidence and severity of infection is increased in pregnancy.
Symptoms include fever, headache, malaise, backache, abdominal pain, pharyngitis
or conjunctivitis. Diagnosis is by blood cultures or placental/neonatal
swabs. Treatment is with high dose penicillin. Listeria infection during
pregnancy can lead to miscarriage, stillbirth, preterm delivery and neonatal
listeriosis, which carries 50% mortality.
Parvovirus B19 infection is spread by respiratory droplets with an incubation
period of 18 days. It is often seen in outbreaks at schools and manifests in
children as erythema infectiosum – a ‘slapped cheek’ appearance. Erythema
infectiosum is also known as the fifth disease – so-called because it is the fifth
of the classical childhood skin rashes (the others being measles, chickenpox,
rubella, scarlet fever and roseola infantum). In adults, parvovirus infection
presents with fever, malaise and arthralgia. In women with parvovirus infection
fetal death is seen in 9%. The second trimester holds the highest risk of fetal
infection. The fetal sequelae are non-immune hydrops due to chronic haemolytic
anaemia and myocarditis. In utero blood transfusion of hydropic fetuses
may prevent demise. There are no long-term sequelae among survivors.
Parvovirus B19 is so-called since it was first discovered in well B19 (row B,
column 19) of a large series of Petri dishes.

How well did you know this?
1
Not at all
2
3
4
5
Perfectly
14
Q

The midwife is delivering a term baby. The head has been delivered.
Which movement should the midwife wait for before delivering the
shoulders?

A. Descent

B. Extension

C. External rotation

D. Flexion

E. Internal rotation

A

C – External rotation

Labour describes the expulsion of the fetus and the placenta by the uterus. There
are regular painful uterine contractions associated with dilation and effacement
of the cervix. A normal spontaneous labour should occur within 12 hours and is
divided into three stages.
The first stage is divided into a latent and active phase. In the latent phase there
is cervical effacement and dilation to 3 cm. The active phase is from 3 cm
dilation to 10 cm. The latent phase is a slow process and may take hours to
days, however the active phase is faster and should only take hours. The
second stage is from full dilation to delivery of the fetus. The passive phase of
the second stage is from full dilation until the woman feels an urge to push as
the head reaches the pelvic floor. The active phase of the second stage is
where the pressure on the pelvic floor generates an irresistible desire to push.

How well did you know this?
1
Not at all
2
3
4
5
Perfectly
15
Q

Which of the following hormones stimulate the growth of primary
follicles?

A. Activin

B. Follicle-stimulating hormone

C. Inhibin

D. Oestrogen

E. Progesterone

A

B – Follicle-stimulating hormone

Menstruation is controlled by the hypothalamic–pituitary–ovarian axis. There
are two phases: the follicular phase (days 1 to 14) and the luteal phase (days
15 to 28). During these phases there are changes in hormone levels, in endometrial
thickness and in cervical mucus.
Follicle-stimulating hormone (FSH) is a glycoprotein produced by the anterior
pituitary gland in response to gonadotrophin-releasing hormone (GnRH) from
the hypothalamus. The concentrations of FSH and luteinizing hormone (LH)
are allowed to rise during early follicular phase due to the low levels of oestrogen
and progesterone at the end of the previous cycle. The action of FSH along with
LH is to stimulate the growth of 6–12 primary follicles each month during the
follicular phase of the cycle (days 1 to 14). As the follicles mature there is a
rise in oestrogen due to increased production from the granulosa cells of the
developing follicles and this increase in oestrogen inhibits the release of FSH
and LH (negative feedback). This mechanism avoids hyperstimulation of the
ovary and the resultant maturation of multiple follicles. Thus only one of these
follicles will reach full maturation at the mid-follicular phase with the others
undergoing atresia.
As the oestrogen levels continue to rise throughout the follicular phase there is a
mild surge in FSH mid-cycle due to the very high levels of oestrogen (positive
feedback) but more importantly there is a concurrent surge in LH which initiates
ovulation. The gonadotrophins LH and FSH are low in the luteal phase and do
not rise again until degeneration of the corpus luteum with subsequent decrease
in steroid hormones (oestrogen and progesterone) at days 26 to 28 to repeat
the cycle.
FSH is low during childhood, increases with puberty to become cyclical
throughout a woman’s reproductive lifetime and is very high after menopause
as there are no follicles left to be released so there is no negative feedback
mechanism from oestrogen and progesterone.

How well did you know this?
1
Not at all
2
3
4
5
Perfectly
16
Q

The obstetric registrar is called to a 28-year-old primigravida who went into
spontaneous labour at 39 þ 5 weeks. She is now fully dilated after having an
epidural and oxytocin. She has been actively pushing for 2 hours and is
exhausted and tells you she cannot push any longer. On examination,
there is no head palpable abdominally, and an occipitoanterior position
is felt vaginally. The station is þ1 with a small caput and no moulding.
How should the baby be delivered?

A. Elective Caesarean section

B. Emergency Caesarean section

C. Kielland’s forceps

D. Manual rotation followed by Neville Barnes forceps

E. Neville Barnes forceps alone

A

E – Neville Barnes forceps alone

Delay in the second stage of labour, if birth is not imminent after 2 hours in
nulliparous women, or one hour in parous women, should be managed by a
trained obstetrician. Forceps are made of two interlocking blades which fit
around the baby’s head to guide it out. When correctly applied they should
lock together easily and not be forced. Neville Barnes forceps have both a
cephalic curve to fit the baby’s head and a pelvic curve which follows the
contours of the sacral hollow. These are used to aid delivery when rotation is
not required such as in this case.

How well did you know this?
1
Not at all
2
3
4
5
Perfectly
17
Q

A 31-year-old primigravid woman with a body mass index of 31 had a positive
glucose tolerance test at 28 weeks consistent with gestational diabetes
mellitus. Although she was advised to change her diet she did not do this
and her glucose control has been suboptimal. An ultrasound scan demonstrated
macrosomia.
Which emergency condition does this put her at a greater risk of?

A. Amniotic fluid embolisation

B. Disseminated intravascular coagulation

C. Shoulder dystocia

D. Uterine inversion

E. Uterine rupture

A

C – Shoulder dystocia

Shoulder dystocia describes impaction of the fetus’s anterior shoulder behind
the symphysis pubis after delivery of the head, impeding delivery of the rest
of the body.
This woman has gestational diabetes mellitus and is more likely to develop
macrosomia. Macrosomia – an abnormally large fetus – occurs in maternal diabetes
due to excessive production of fetal insulin and an increased deposition of
glycogen in the fetus. Macrosomia increases the risk of shoulder dystocia due to
the size of the shoulders. Other risk factors for shoulder dystocia include a past
history of dystocia, maternal obesity, prolonged first stage of labour, secondary
arrest .8 cm cervical dilation, mid-cavity arrest and forceps/ventouse delivery.
Consider shoulder dystocia if the head delivers slowly or with difficulty and
the neck does not appear, or if the chin retracts against the perineum (the
turtle sign).
Fetal morbidity and mortality occurs due to compression of the umbilical cord
between the fetal trunk and the maternal pelvis which rapidly leads to fetal
hypoxia and death. Nerve injury can also occur due to downward traction on
the head during attempts to deliver the baby (! Erb’s palsy or Klumpke’s
palsy). Fracture of the fetal humerus or clavicle may also occur but may be
necessary for delivery. Maternal complications include birth canal injuries,
femoral nerve injury from excessive hip flexion and increase in maternal
blood loss.

How well did you know this?
1
Not at all
2
3
4
5
Perfectly
18
Q

A 32-year-old woman complains of longstanding painful, heavy periods.
She has had two normal vaginal deliveries after difficulty conceiving with
both pregnancies. She suffers from significant pain on intercourse. On
further questioning she also states she has had occasional rectal bleeding
during her menstrual cycle throughout her life. Her past history includes
an appendicectomy aged 10. Pelvic examination reveals a fixed retroverted
uterus that is tender.
What is the most likely explanation for her pain?

A. Adhesions from surgery

B. Chronic pelvic inflammatory disease

C. Endometriosis

D. Fibroid degeneration

E. Ovarian cyst

A

C – Endometriosis

Endometriosis is the most likely diagnosis, with the typical symptoms of abdominal
pain, dyspareunia, secondary dysmenorrhoea and subfertility. Bimanual
palpation in this case has revealed a tender fixed uterus but there can also be uterosacral nodules, endometriomas, uterine or ovarian enlargement or adnexal
tenderness.
Endometriosis is where functioning endometrial tissue is seen outside the cavity
of the uterus. This tissue responds to the cyclical hormonal changes and bleeds
at the time of menstruation forming abdominal adhesions. The ovaries, uterosacral
ligaments or ovarian fossae are common sites of deposition of this
endometrial tissue. If there is deposition near the bowels, rectal bleeding can
be seen. It is most common in 25 to 35-year-olds. The aetiology is unclear
and likely to be multifactorial.
An ultrasound is useful to exclude other pathology but is not diagnostic of
endometriosis. It may pick up an endometrioma (blood filled ovarian cysts
often seen in endometriosis). Endometriomas are also known as chocolate
cysts due to their dark reddish brown appearance. Diagnosis can only be
made by clinical appearances on laparoscopy. Treatment choices must involve
the patient as it is dependent on age, fertility wishes and location and severity
of disease. Medical treatment includes analgesia, the combined oral contraceptive
pill, progestogens, gonadotrophin-releasing hormone analogues and
antiandrogens. Although useful for symptomatic relief many of these options
are not appropriate if the woman is trying to conceive. Surgical treatment is
used where fertility is required. Infertility is common even without visible tube
blockage. In vitro fertilization is commonly indicated.
Chronic pelvic inflammatory disease describes episodes of recurrent acute pelvic
infection which result in chronic inflammation of the pelvic organs with multiple
adhesions causing abdominal pain, dyspareunia and subfertility. Chronic pelvic
pain can be caused by adhesions from previous surgery however the significant
menstrual symptoms again point you away from this diagnosis. Treatment
options for this include adhesiolysis; however, this is not proven and is more
likely to be effective if there are large adhesions and if it is done laparoscopically.
There is still a significant risk of visceral damage and a chance of further
adhesions developing.
The pain from an ovarian cyst can be due to torsion, cyst rupture or bleeding
into a cyst. This would be unlikely to give menstrual and fertility problems.
Other causes of chronic pelvic pain can be functional pain, constipation or irritable
bowel syndrome.

How well did you know this?
1
Not at all
2
3
4
5
Perfectly
19
Q

A 43-year-old woman has conceived naturally for the first time despite two
failed IVF attempts. She is very concerned about Down’s syndrome and
would like a test performed as soon as possible that would give a firm
diagnosis. She is currently at 11 weeks gestation.
What test would be most appropriate?

A. Amniocentesis

B. Chorionic villus sampling

C. First trimester ultrasound scan

D. Nuchal translucency test

E. Serum triple test

A

B – Chorionic villus sampling

Chorionic villus sampling is a diagnostic test which means a definite diagnosis
can be given rather than a ‘risk’ of a condition being present. A biopsy of the
chorionic villus is taken either transabdominally or transcervically under ultrasound
guidance. Cells obtained are analysed and a result is ready in 48 hours.
The risk of miscarriage is around 1–2% and couples must be counselled
appropriately before undertaking a test like this due to its invasive nature. It is
performed at 11 to 14 weeks which means this woman could proceed with
this test after appropriate counselling. The early gestation means a decision
regarding termination of pregnancy can be made as early as possible before a pregnancy becomes easily visible under clothes. Chorionic villus sampling is not
performed before 9–11 weeks as fetal limb abnormalities can occur. There are
cases where inconclusive results can occur due to placental mosaicism; this
would mean that amniocentesis would have to be performed later. There can
be maternal contamination which would lead to false-negative results. Rhesus
D negative women must receive anti-D immunoglobulin.
Nuchal translucency is a screening tool for Down’s syndrome and other abnormalities.
It is used at 11–14 weeks. A measurement is made of the thickness of the
skin fold over the neck of the fetus. This information is combined with the
mother’s age and gestation of the pregnancy. This will not provide a definite
diagnosis. A first trimester ultrasound scan is used to establish viability and
confirm gestation during the first trimester. It detects defects in gross
anatomy and determines the chorionicity and amnionicity of multiple pregnancies.
It gives limited other information and on a routine scan no definitive
diagnosis of Down’s syndrome can be made.

How well did you know this?
1
Not at all
2
3
4
5
Perfectly
20
Q

A 26-year-old woman with a twin pregnancy has developed twin-to-twin
transfusion syndrome at 34 weeks gestation.
What type of twins is she likely to have?

A. Dizygotic dichorionic diamniotic

B. Dizygotic dichorionic monoamniotic

C. Dizygotic monochorionic monoamniotic

D. Monozygotic dichorionic diamniotic

E. Monozygotic monochorionic diamniotic

A

E – Monozygotic monochorionic diamniotic

The incidence of twins is 1 in 100, and triplets 1 in 1000. Predisposing factors are
increasing maternal age and parity, personal or family history, race and assisted
conception (20% of in vitro fertilization pregnancies are multiple). Multiple
pregnancies need closer monitoring as fetal and maternal mortality and morbidity
are greater. The mother has a greater risk of hyperemesis, miscarriage, hypertension
and pre-eclampsia, gestational diabetes, polyhydramnios (especially
with monozygotics), anaemia, antepartum and postpartum haemorrhages
and placenta praevia. The fetal risks include increased perinatal mortality,
increased congenital abnormality, preterm labour, placental insufficiency or
intrauterine growth restriction (especially in monozygotics), malpresentation,
twin–twin transfusion and vanishing twin syndrome.
Twin-to-twin transfusion is where, due to anastomosis of vessels in the single
placental mass of a monochorionic twin pregnancy, one twin gains at the
other’s expense. One twin becomes anaemic, hypovolaemic, oligohydramniotic
and growth-restricted while the other one develops polycythaemia, hypervolaemia,
polyuria and polyhydramnios. It occurs in varying degrees in up to 35% of
monochorionic twins and accounts for 15% of perinatal mortality. Ultrasound
scan is used to look at fetal wellbeing and to identify any abnormalities such
as liquor volume that may suggest twin-to-twin transfusion. Therapeutic amniocentesis
may be used to reduce the amniotic fluid pressure. Laser ablation of
placental vessels can be useful although there are risks of fetal demise or congenital
abnormalities. Vanishing twin syndrome is where a fetus in a multiplegestation
pregnancy dies in utero and is subsequently reabsorbed by the
mother (either partially or completely).

How well did you know this?
1
Not at all
2
3
4
5
Perfectly
21
Q

A 32-year-old woman is 40 weeks þ 6 days and is having induction of
labour for mild pre-eclampsia. She has had a total of 6 mg prostaglandin
tablets per vagina. On abdominal examination the head is 5/5 palpable
and on vaginal examination she is 3 cm dilated with intact membranes
and a station of –3. She has mild contractions. The plan is to undertake
an artificial rupture of membranes.
Which emergency condition does this put her at a greater risk of?

A. Cervical shock

B. Cord prolapse

C. Shoulder dystocia

D. Uterine inversion

E. Uterine rupture

A

B – Cord prolapse

Cord presentation is where the cord lies below the presenting part with intact
membranes, meaning it is more likely to prolapse when the waters break. This
is life-threatening for the baby as the supply of blood can be greatly reduced
due to either mechanical compression of the umbilical cord by the presenting
part or spasm of the umbilical vessels due to cooling, drying, pH change
and handling. The cardiotocograph (CTG) will reflect the effect of these
changes by indicating fetal distress, showing deep decelerations or a prolonged
bradycardia.
There is an increased risk of cord prolapse in artificial rupture of membranes
(ARM) due to the sudden rush of fluid particularly with a high presenting
part. Therefore it is important to perform a vaginal examination following an
ARM to ensure there is no cord prolapse. In this case it may be prudent to perform the ARM in theatre so an emergency Caesarean section could be
performed immediately if the cord did prolapse.
As seen in this case, cord prolapse is more likely in cases where there is not a
close fit between the presenting part and the pelvic inlet such as a high head,
malpresentation such as transverse or oblique lie, breech presentation particularly
footling breech, prematurity, polyhydramnios and fetal growth restriction.
It is also seen where there is a long umbilical cord or in a 2nd twin. Of
course it can occur without risk factors and a vaginal examination should be
done if there are CTG abnormalities after spontaneous rupture of membranes
to rule out a cord prolapse.
Management as with all emergencies is to call for help including senior obstetricians,
anaesthetist and alert theatre staff as it is important to deliver the baby as
soon as possible. Stop any oxytocin infusions. If the woman is fully dilated
forceps or ventouse delivery should be attempted and if she is not fully
dilated an emergency Caesarean section (category 1) should be performed.
On the way to theatre, pressure must be taken off the cord so a head down position
is adopted on the bed with the woman placed on all fours with her buttocks
uppermost. A hand can be placed in the vagina to lift the presenting
part off the cord to prevent compression. The maternal bladder can be filled
with saline which keeps the presenting part off the cord.

22
Q

You are asked to examine a woman’s perineum after she has delivered
vaginally. You see that the perineal skin and muscles are torn. On
examination of the anus, the external anal sphincter is torn but less
than 50% so.
What degree of perineal tear has this woman sustained?

A. 1st degree tear

B. 2nd degree tear

C. 3rd degree tear 3a

D. 3rd degree tear 3c

E. 4th degree tear

A

C – 3rd degree tear 3a

Tears of the perineum can occur in normal deliveries but are often more severe in
operative delivery particularly if an episiotomy is not performed.
Tears are graded as follows:
† First degree – injury to perineal skin only. There is no need for routine
suturing unless haemostasis is a problem
† Second degree – injury to perineum involving perineal muscles only
without involvement of the anal sphincter. These should be sutured to
ensure correct apposition of the perineal muscles and skin and to secure
haemostasis. Many midwives will suture second degree tears
† Third degree – injury to perineum involving the anal sphincter complex.
These must be sutured in theatre with adequate analgesia, spinal or epidural
top-up, by a trained surgeon normally the obstetric registrar or consultant.
The anal sphincter must be repaired to avoid incontinence
W 3a – less than 50% external anal sphincter (EAS) thickness torn
W 3b – more than 50% EAS thickness torn
W 3c – both EAS and internal anal sphincter (IAS) torn
† Fourth degree – injury to perineum involving the anal sphincter complex
and anal epithelium. These must be repaired in theatre by a trained obstetrician
with the assistance of a general surgeon if required

23
Q

Which hormone, produced by the corpus luteum, is low in the follicular
phase and maximal in the luteal phase?

A. Follicle-stimulating hormone

B. Inhibin

C. Luteinizng hormone

D. Oestradiol

E. Progesterone

A

E – Progesterone

Progesterone is a steroid hormone produced by the corpus luteum. The corpus
luteum is the remnant of the Graafian (ovarian) follicle after the oocyte and
cumulus oophorus have been expelled. After ovulation the remnants are penetrated
by capillaries and fibroblasts while the granulosa cells undergo luteinization
to collectively form the corpus luteum (yellow body).
There are very low levels of progesterone during the follicular phase. There is an
immediate rise in progesterone following ovulation due to the formation of the
corpus luteum and progesterone concentration is maximal in the luteal phase of
the menstrual cycle (days 15 to 28). If conception does not occur the corpus
luteum will degenerate after 14 days and production of both steroid sex hormones
declines and the next cycle begins. If conception and implantation
occur the corpus luteum is maintained by gonadotrophin released by the
trophoblast.
The luteal phase of the menstrual cycle correlates to the secretory phase of
the endometrium. The high level of progesterone means secretory vacuoles
develop in the glandular epithelium below the nuclei, which then release
secretions into the lumen of the glands which become tortuous. After ovulation
the rise in progesterone opposes the action of oestrogen and stimulates
production of thick cervical mucus, which is less penetrable by sperm, and
contraction of the cervical os.

24
Q

A 29-year-old woman is in spontaneous labour at 40 weeks. The CTG is reactive.
She has had one previous lower segment Caesarean section. On
abdominal examination, the head is 0/5 palpable. On vaginal examination,
she has been 10 cm dilated for 20 minutes. There is an occipitoanterior
presentation with a flexed head. The head is at station þ1 and there is no
caput and no moulding.
How should the baby be delivered?

A. Elective Caesarean section

B. Emergency Caesarean section

C. Kielland’s forceps

D. Normal delivery

E. Neville Barnes forceps

A

D – Normal delivery

This case describes very good conditions for a normal delivery. Most importantly
the cardiotocograph is normal so labour can continue without intervention.

The head is low in the pelvis, as shown by there being no head palpable per
abdomen and a station of þ1, which increases the chances of a normal delivery.
Abdominal examination of the fetal head is described by ‘fifths’ of the head
palpable. The head can be 5/5 palpable where all the head can be felt per
abdomen and if it is ‘free’ then it can be balloted between the examiner’s
hands. The head is described as engaged if there is less than 2/5 palpable. In
this case the head cannot be palpated abdominally so is described as 0/5
palpable. Station describes the position of the presenting part in relation to
the ischial spines. A station of 0 is where the presenting part is level with the
ischial spines. A station above the ischial spines is described as –1 to –3, and
below the ischial spines (closer to the introitus) is described as þ1 to þ2. A
vaginal delivery is more likely with a more positive station.
She has reached full dilation and a flexed head in the occipitoanterior is the
optimal position of the fetal head to achieve a normal delivery. There is no
caput or moulding which indicates there is adequate room in the pelvis for
the head to descend as there has been minimal compression on the fetal head.
Vaginal delivery is not contraindicated following one Caesarean section and is
known as VBAC (Vaginal Birth After Caesarean). There is increased monitoring
and more cautious use of induction and augmentation agents in these cases.

25
Q

A 31-year-old pregnant woman was involved in a minor road traffic
collision where she banged her abdomen on the steering wheel. Serious
injury has been excluded but she is concerned about the baby. She has
good fetal movements and has had no bleeding per vagina. The fetal
heart is heard and is regular. She is 25 weeks gestation and is RhD negative.
She has had no previous children.
What action needs to be taken with regards to anti-D prophylaxis?

A. Give antenatal anti-D prophylaxis 250 iu

B. Give antenatal anti-D prophylaxis 500 iu

C. Give postnatal anti-D

D. Give routine antenatal anti-D prophylaxis at 28 weeks

E. No action needed at present

A

B – Give antenatal anti-D prophylaxis 500 iu

At booking all women are offered blood tests to screen for infection and to
establish maternal blood group. Fifteen percent of women are rhesus negative
(RhD –ve) which means their red blood cells lack the rhesus D antigen
(compared to RhD þve women who do carry the D antigen). The rhesus
status of a woman must be clearly documented on her antenatal notes as if
there is any passage of fetal blood into the maternal circulation there can be
serious consequences for subsequent pregnancies.
The lack of rhesus D antigen in RhD –ve women means they are at risk of developing
anti-D antibodies to an RhD þve fetus. If any fetal blood cells from an
RhD þve fetus cross into the circulation of an RhD –ve woman she will react
to the ‘foreign’ anti-D antigens on the fetus’s red blood cells and produce antibodies.
This is known as a sensitizing event. This is of no consequence in the
current pregnancy but in future pregnancies the maternal anti-D antibodies
can cross the placenta and destroy the blood cells of a subsequent RhD þve
fetus resulting in haemolytic disease of the newborn (varies from subclinical
disease to severe disability or death). Anti-D prophylaxis prevents production
of anti-D antibodies by providing anti-D immunoglobulins either routinely or
after a sensitizing event. The frequency of haemolytic disease of the newborn
was 1% of all births before immunoprophylaxis. Now it is reduced to 1 in
21,000 births. Anti-D immunoglobulin is produced from plasma collected
from male donors purposefully sensitized.

26
Q

You examine a lady who has attended a labour ward for induction of labour
at term þ 12 days. She has a cephalic presentation which is three-fifths
palpable. The cervix is not dilated at all, is 3 cm long, of average consistency
and is in a mid-position. The station is –2.
What is her Bishop score?

A. 0

B. 2

C. 4

D. 6

E. 8

A

C – 4
The Bishop score is a classification system used to describe the ‘favourability’ of
the cervix. A higher score is associated with an easier, shorter labour that is less
likely to fail. Although it is a universal system it is subject to examiner variation.
Therefore the same person should examine the woman to assess progress.

The modified Bishop scoring system is as follows:

Cervical feature 0 1 2 3

Dilation (cm) ,1 1–2 2–4 .4

Consistency Firm Average Soft –

Length of cervix (cm) .4 2–4 1–2 ,1

Position Posterior Mid/Anterior – –

Station of presenting
part (related to
ischial spines)
–3 –2 –1 or 0 Below spines

This woman scores 0 for dilation, 1 for length of cervix, 1 for station, 1 for consistency
and 1 for position, giving a score of 4.
Labour is unlikely to be imminent if the score is less than 6 so induction with
vaginal prostaglandins would be indicated. If the score is above 9, labour is
likely to progress without medical assistance although a stretch and sweep
may speed up the process or if possible an artificial rupture of membranes
could be performed. At a score between 6 and 9, management will depend
on the indication for induction.

27
Q

A 43-year-old woman has recently been diagnosed with cervical cancer.
Which of the following are risk factors for the development of cervical
cancer?

A. Early menarche

B. Early menopause

C. Increased number of sexual partners

D. Nulliparity

E. Progesterone-only pill

A

C – Increased number of sexual partners

Cervical cancer is largely a disease of sexually active women. Women with the
disease have generally had more sexual partners, are likely to have started
having intercourse earlier and are less likely to have used barrier protection.
(Always remember that the sexual behaviour of their partners is also important.)
The reason for this correlation is that cervical cancer has a strong association
to the human papillomavirus (HPV), a sexually transmitted DNA virus. There
is a link between HPV serotypes 16, 18 and 33 with pre-invasive disease and
invasive cancer.
Other risk factors include:
Oral contraceptive pill. There is some evidence to suggest that prolonged use of
the pill produces a moderately increased risk of cervical cancer of both the squamous
cell and rarer adenocarcinoma type. There is also an associated increase
in mortality. It has been suggested that this effect may be due to differences
in sexual behaviour in women who use the oral contraceptive pill compared
with those who do not, however there is no evidence demonstrating that the
progesterone-only pill increases cervical cancer risk. Such studies cannot be
corrected for all variables as it is difficult to establish the exact sexual activities
of both the women and their partners.
Smokers are at increased risk of cervical cancer. This may be due to chemical carcinogenesis
or to alterations in immune function in the cervical epithelium as
smoking is known to reduce the number of Langerhans’ cells which are involved
in local immune surveillance. Immunocompromised women are more likely to
contract cervical cancer.
Unlike endometrial carcinoma, there is no link to the onset or arrest of menstruation
in cervical cancer. Parous women are more likely to suffer from cervical
carcinoma.

28
Q

A 39-year-old primigravid woman who has had no antenatal care attends a
labour ward at 39 weeks gestation. She complains of heavy, unprovoked,
painless vaginal bleeding. On examination, she has a soft non-tender
abdomen and the head is not engaged. She is passing clots per vagina.
She has a pulse rate of 112/min and her blood pressure is 96/56 mmHg.
The CTG is non-reassuring.
What is the most likely diagnosis?

A. Cervical ectropion

B. Placental abruption

C. Placenta praevia

D. Uterine rupture

E. Vasa praevia

A

C – Placenta praevia

Antepartum haemorrhage is defined as bleeding after 24 weeks gestation.
Between 2 and 4% of women have antepartum haemorrhage. Bleeding can
be maternal, fetal or placental. Sometimes no cause can be obvious.
Placenta praevia is where the placenta lies close to, or covers, the internal os. It
occurs in 1% of pregnancies. It is more common in older women, previous
Caesarean section, increasing parity, previous placenta praevia, smoking and multiple gestations.

A placenta praevia may be found at the 20-week scan and, if seen, a repeat
scan at 34–36 weeks is performed. Ninety percent of placenta praevia seen at
the 20-week scan will no longer be praevia later on as the placenta moves
away from the os due to uterine growth.
Clinically, there is sudden-onset painless unprovoked or postcoital fresh red
blood per vagina in the second or third trimester. On abdominal examination
there is a soft non-tender uterus with a high head or malpresentation as the
placenta blocks the passage of the presenting part into the pelvis. The clinical
condition of the mother correlates with the visible blood loss (unlike uterine
abruption). Maternal blood is lost so there is little risk to the fetus unless the
mother becomes hypovolaemic.
Management depends on multiple factors including gestation of the fetus and
clinical condition of the mother. There could be sudden major haemorrhage at
any time. For this reason some women who have had heavy bleeds or who live
far from the hospital are advised to be admitted from 30–32 weeks until delivery
(which is generally planned for 38–39 weeks but may be earlier if indicated). If
there is major haemorrhage that is threatening maternal or fetal life, a Caesarean
section should be performed. Risks include haemorrhagic shock, complications
of Caesarean section, transfusion-related complications and, if there is a concurrent
placenta accreta, massive haemorrhage (which may need hysterectomy).

29
Q

A 23-year-old woman with a positive pregnancy test complains of lower
abdominal cramping with what she describes as a period-type bleed at
home. On abdominal palpation there is mild suprapubic discomfort. On
speculum examination a small amount of blood is seen in the vagina and
the cervical os is closed. Urine dipstick is unremarkable.
What is the most likely diagnosis?

A. Complete miscarriage

B. Incomplete miscarriage

C. Inevitable miscarriage

D. Menstruation

E. Threatened miscarriage

A

E – Threatened miscarriage

Miscarriage is defined, in the UK, as the loss of pregnancy before 24 weeks. It
is the most common complication of pregnancy and occurs in 25% of all pregnancies.
The risk of miscarriage is highest early in pregnancy and decreases as
gestation increases. Underlying causes of miscarriage include fetal abnormality
(50%), infection (ToRCH – Toxoplasmosis, other infections, Rubella virus, Cytomegalovirus
and Herpes simplex virus), increasing maternal age, maternal
illness (diabetes, renal disease), abnormal uterine cavity (intrauterine device
insertion, congenital septum), antiphospholipid syndrome and intervention
(amniocentesis and chorionic villus sampling). It is important to inform
women that exercise, intercourse and emotional trauma do not cause miscarriage.
Investigation into the cause of miscarriage is only performed if three miscarriages have occurred. Although miscarriages are very common they are
still distressing to the parents so it is important to act with care and sensitivity.
Clinical presentation of miscarriage is varied with some women experiencing
very little pain and bleeding and easily passing the fetus while some women
have life-threatening vaginal bleeding.
This scenario describes a threatened miscarriage. There can be cramping lower
abdominal pain with vaginal bleeding. On examination the cervical os is
closed. Only 25% of threatened miscarriages eventually miscarry. A scan
should be arranged to investigate the bleeding and to confirm the viability of
the pregnancy. This can be done in an early pregnancy assessment unit.
An inevitable miscarriage generally presents with cramping abdominal pain and
vaginal bleeding but on examination the cervical os is open in contrast to a
threatened miscarriage. The fetus may still be alive, but miscarriage will occur
due to dilation of the cervical os. An incomplete miscarriage is where some
fetal material has been passed, but some products of conception are retained
in the uterus (these are visible on ultrasound scanning). The cervical os
remains open until all the products have passed (whether that be spontaneous,
or with medical or surgical assistance). This can be associated with a significant
amount of bleeding. A complete miscarriage describes passage of all fetal tissue
with closure of the cervical os. Hospital follow-up is generally not required.
Finally, a missed miscarriage (also known as a delayed or silent miscarriage) is
when the fetus dies in utero and the cervix stays closed and is only discovered
on an ultrasound scan. There may or may not be bleeding in this case. The
management for this can be conservative, medical or surgical.

30
Q

A 34-year-old woman who has had four previous normal deliveries attends
the antenatal day unit at 40 weeks complaining of abdominal pain associated
with fevers and sweating. Initially she thought she was in labour as
she had spontaneous rupture of membranes four days ago, but she says
the pain has become increasingly worse. On examination, her uterus is
very tender and the cervix is long and closed with an associated yellow–
green discharge. Her temperature is 39.78C. She has raised white cell
count and C-reactive protein.
What is the likely diagnosis?

A. Abruption

B. Acute pyelonephritis

C. Chorioamnionitis

D. Uterine rupture

E. Urinary tract infection

A

C – Chorioamnionitis

Chorioamnionitis is infection of the amniotic cavity and the chorioamniotic
membranes. Causative pathogens include Escherichia coli, Streptococcus
and Enterococcus faecalis. Both mother and fetus can develop potentially lifethreatening
septicaemia. It is normally seen after rupture of membranes particularly
if this has been over a prolonged period of time, although it can occur
with intact membranes. Other risk factors include prolonged labour, preterm
labour, internal fetal monitoring, cervical examinations or urinary tract or
vaginal infection.
A woman with chorioamnionitis presents with abdominal pain, uterine tenderness,
maternal pyrexia and tachycardia, raised C-reactive protein and white cell
count, meconium or foul smelling liquor. There could be fetal tachycardia or a
non-reassuring cardiotocograph. If there is any suspicion of chorioamnionitis
the baby must be delivered without delay (ideally by Caesarean section unless
labour is well-established). Antibiotics are needed for the mother and baby
and the paediatricians should be present at the delivery

31
Q

A 33-year-old woman attends the emergency department complaining of
an aching pain in her left iliac fossa. This has been present intermittently
for a few months. She says the pain is significantly worse today but
remains focused in the left iliac fossa. She has vomited four times. She
denies being sexually active. On examination, she is tender in the left
iliac fossa with some voluntary guarding. Speculum examination revealed
no abnormalities. There is left adnexal tenderness on vaginal examination
but no cervical excitation. Her observations show heart rate 112/min, blood
pressure 98/62 mmHg and temperature 36.88C. A urine result is awaited.
What is the most likely diagnosis?

A. Appendicitis

B. Mittelschmerz

C. Ovarian cyst torsion

D. Pelvic inflammatory disease

E. Renal colic

A

C – Ovarian cyst torsion

This case describes an intermittent ovarian cyst torsion which has become an
acute problem. Torsion of an ovarian cyst results in a sudden-onset severe, unilateral,
colicky or twisting pain with intermittent pain if torsion is incomplete.
Vomiting is common and tachycardia, hypotension and pyrexia are found on
examination. This must not be missed as delay can lead to irreversible ischaemia
of the ovary. An urgent laparoscopy is needed where a cystectomy may be completed,
but signs of ischaemia or necrosis warrant oophorectomy.
The management of untorted ovarian cysts vary depending on symptoms and
investigation findings. For a cyst to be low risk the patient would have a
normal serum CA-125 and ultrasound would show a simple, unilateral, unilocular
cyst under 5 cm. These can be managed conservatively if the patient is
asymptomatic, as 50% resolve spontaneously, but must be followed up in 3
to 4 months. If cysts are symptomatic then surgical removal is considered. A
cyst with an elevated CA-125, with ultrasound appearances of a complex,
bilateral or multinodular cyst, or that is larger than 5 cm is considered to be
high-risk. Surgical removal may be needed in a specialist centre. A histological
diagnosis is essential for appropriate follow-up.

32
Q

A 31-year-old woman, who has recently had a positive pregnancy test, has
an early pregnancy transvaginal scan which showed no intrauterine pregnancy.
She subsequently has two bhCG samples taken, the first on the
day of the scan and the second 48 hours later. The first result was 578 iu
and the second result was 1126 iu.
What is the most likely diagnosis?

A. Ectopic pregnancy

B. Early intrauterine pregnancy

C. Inevitable miscarriage

D. Missed miscarriage

E. Threatened miscarriage

A

B – Early intrauterine pregnancy

The fact that the bhCG was below 1000 iu on the day of the transvaginal scan
explains the reason why an intrauterine pregnancy was not seen. Generally a
level of 1000 iu or more is associated with being able to visualize the gestational
sac. The bhCG was therefore repeated 48 hours later and doubled, suggesting a
healthy intrauterine pregnancy. An ultrasound scan can be repeated in a week to
confirm viability of the pregnancy.
If the repeat bhCG is static or only slowly reducing, an ectopic pregnancy must
be ruled out and further management should be considered. If the bhCG level
rapidly reduces it is likely that the pregnancy is aborting and the patient can be
treated conservatively with regular bhCG measurements to ensure resolution.
An empty uterus can mean three things: a complete miscarriage, very early
intrauterine pregnancy that is too small to visualize, or an ectopic pregnancy.
As seen in this question, bhCG can be used to aid diagnosis. If bhCG is
.1000 iu a gestational sac should be seen on a transvaginal scan, so an
empty uterus with this result could indicate an ectopic pregnancy. If bhCG is
,1000 iu it may be too early to visualize a gestational sac and serial bhCG
must be used. Serum bhCG should double in 48 hours in viable intrauterine
pregnancies. A plateau of bhCG suggests ectopic pregnancy and a significant
fall in bhCG suggests a miscarriage, however this is not always accurate. An
ultrasound can be repeated in 1 week if the bhCG doubles and the patient
remains well. A diagnostic laparoscopy is considered if the bhCG levels are
not rising satisfactorily.

33
Q

A 23-year-old woman complains of intense cramping pains that start on the
second day of her period and last for 2 days. She has experienced these since
menarche. She is otherwise fit and well.
What is the diagnosis?

A. Primary amenorrhea

B. Primary dysmenorrhoea

C. Primary menorrhagia

D. Secondary dysmenorrhoea

E. Secondary menorrhagia

A

B – Primary dysmenorrhoea

Dysmenorrhoea describes cyclical cramping pains occurring before or during
menstruation, which may be associated with malaise and gastrointestinal
symptoms. Fifty percent of women have some pain during periods, with 10% describing it as severe. Primary dysmenorrhoea occurs from menarche and there
is often no cause found. Secondary dysmenorrhoea has an onset after menarche
and is often associated with pathology such as endometriosis, pelvic inflammatory
disease, fibroids or iatrogenic causes including intrauterine devices or
cervical stenosis after large loop excision of the transformation zone (LLETZ).
Treatment for both primary and secondary dysmenorrhoea includes hot water
bottle, transcutaneous electrical nerve stimulation, acupuncture and nonsteroidal
anti-inflammatory drugs (NSAIDs) especially mefanamic acid (these
reduce the uterine production of prostaglandins and can also reduce heavy
blood loss). Hormonal treatments include combined oral contraceptives,
depot progestogens and the Mirena coil (levonorgestrel intrauterine device).
If these provide no relief, laparoscopic uterosacral nerve ablation can be
considered.
Amenorrhoea is the absence of menstruation. This would be a normal occurrence
before puberty, during pregnancy and milk secretion and after menopause.
Primary amenorrhoea means the menstrual cycle never begins due to
anatomical abnormalities, genetic disorders or hormonal imbalances. Secondary
amenorrhoea means menstruation stops having previously been present
due to disorders of the hypothalamic–pituitary axis, deficiency of ovarian or
thyroid hormones, mental disturbances, a

34
Q

Which term describes the transition from a left occipitotransverse position
to an occipitoanterior position as the head passes through the pelvis?

A. Effacement

B. Extension

C. External rotation

D. Flexion

E. Internal rotation

A

E – Internal rotation

35
Q

A 25-year-old woman is brought into the resuscitation area having
collapsed. She is maintaining her airway, is breathing with a non-rebreathe
bag and has a weak pulse. The nurse informs you her observations are: heart
rate 125/min, blood pressure 86/48 mmHg, saturations 98%, temperature
37.28C. After fluid resuscitation the patient is responsive enough to tell
you she had some right-sided lower abdominal pain earlier in the day but
cannot remember anything else. She says she uses the copper coil for
contraception. On examination, she is tender with guarding in the right
iliac fossa. On speculum examination there is brown discharge seen in
the vagina. Threads from the coil are seen and cervical excitation is
present. A urine result is awaited.
What is the most likely diagnosis?

A. Appendicitis

B. Dislodged coil to a cervical location

C. Ectopic pregnancy

D. Ovarian cyst torsion

E. Pelvic inflammatory disease

A

C – Ectopic pregnancy

This is most likely to be an ectopic pregnancy which may have ruptured leading
to bleeding into the abdominal cavity causing the woman to collapse and
become clinically shocked.
Ectopic pregnancy describes implantation of the embryo outside the uterine
cavity, most commonly in the fallopian tube, but also in the cornua of uterus,
cervix, ovary or abdominal cavity. Presentation is variable and ectopic must
always be considered in a woman of childbearing age with amenorrhoea,
lower abdominal pain, abnormal vaginal bleeding associated with dizziness,
fainting, shoulder pain or collapse. On examination there is abdominal tenderness
with cervical excitation, tender adnexae or an adnexal mass in a shocked
patient. If the patient is shocked, fluid resuscitation is required followed by
urgent laparoscopy or laparotomy to remove the ectopic and stop the bleeding.
In a stable patient transvaginal and abdominal ultrasound scans are needed.
Depending on the scan result, management can be medical (methotrexate)
with serial bhCG measurements, or surgical. Rhesus D negative women
should be given anti-D.

36
Q

A 29-year-old primigravid woman of 38 weeks þ 5 days gestation attends
the antenatal day unit after suddenly feeling unwell. She has vomited
five times and complains of severe right upper quadrant pain. On examination,
she is tender in the right upper quadrant only. Her blood results
show mild anaemia, low platelets, deranged liver enzymes, and a normal
white cell count. Observations reveal temperature 36.78C, blood pressure
168/96 mmHg and pulse rate 76/min. Her blood glucose level is 5 mmol/L.
What is the most likely diagnosis?

A. Acute fatty liver of pregnancy

B. Acute pyelonephritis

C. Cholecystitis

D. HELLP syndrome

E. Obstetric cholestasis

A

D – HELLP syndrome

HELLP syndrome is the hepatic manifestation of pregnancy-induced hypertension,
characterized by hepatic and haematological dysfunctions. The name is
an acronym of the biochemical findings: Haemolysis, Elevated Liver enzymes
and Low Platelets. The patient presents with nausea, vomiting and epigastric/
right upper quadrant pain due to haemorrhage or distension of the liver
capsule. There is often a sudden onset and, despite the normal association
with pre-eclampsia, 10–20% of women with HELLP have no high blood
pressure and therefore no warning that they will develop the condition.
HELLP has a high mortality and morbidity with progression to acute renal
failure and disseminated intravascular coagulation, and there is increased risk
of abruption. Treatment includes correction of coagulopathy, anti-seizure prophylaxis
and antihypertensives. The fetus must be closely monitored by cardiotocograph
and ultrasound scan if needed. Delivery is the only cure but
deterioration can occur 48 hours after delivery. The woman must be counselled
that the recurrence rate in subsequent pregnancies is 20%.
Acute fatty liver of pregnancy is rare but serious with a maternal mortality of up to
20%. Presentation is similar to cholecystitis with sudden-onset epigastric pain,
anorexia, malaise, nausea, vomiting and diarrhoea. However in acute fatty
liver there is jaundice, mild hypertension or proteinuria, and it can progress to
fulminant liver failure. Biochemically there is raised bilirubin with abnormal
liver enzymes, leukocytosis, thrombocytopenia, hypoglycaemia and coagulation
defects. This is biochemically distinguishable from HELLP syndrome due
to hypoglycaemia and high uric acid. Diagnosis is generally clinical with CT or
MRI but a liver biopsy can be taken if needed. Management is with strict fluid
balance, correction of coagulopathy and electrolyte disturbances and hasty
delivery. Depending on the severity of the condition admission to liver unit or
ICU may be needed.

37
Q

A 38-year-old woman is seen at 29 weeks gestation. She has had one normal
delivery at 41 weeks, one emergency C-section at 30 weeks, two miscarriages
at 10 and 12 weeks and also a termination of pregnancy at 13
weeks with a different partner when she was 16 years old.
How would you describe her gravidity and parity?

A. G2 P6þ3

B. G5 P2þ3

C. G6 P2þ3

D. P3 G5þ2

E. P6 G2þ3

A

C – G6 P2þ3

An obstetric history must include the number of pregnancies a woman has had
and the outcomes, including gestation at delivery, listed in chronological order.
If gestation was over 24 weeks a number of other details must be ascertained
including mode of delivery and, if not a normal delivery, then why, any complications
during pregnancy/delivery to mother/baby, the sex and weight of the
baby and the condition of the baby at birth. Some women have poor obstetric
histories, which can include multiple miscarriages or stillbirths so be prepared to
be sympathetic.
Gravida is the number of times a woman has been pregnant regardless of the
gestation at delivery. The woman in the case has been pregnant six times,
which includes one normal delivery, one Caesarean section, two miscarriages,
one termination and the current pregnancy. This is indicated as G6.
Parity describes the number of potentially viable babies a woman has delivered;
this includes all live births (even before 24 weeks) and stillbirths delivered after
24 weeks. The woman in the question would be a para 2 (P2) as she has had two
deliveries over 24 weeks gestation. The third number preceded by a ‘þ’ indicates
all pregnancies up to 24 weeks which did not result in a live birth so this
includes miscarriages and terminations. A full description of parity would be
para 2þ3 indicating two miscarriages before 24 weeks and one termination.
Nulliparity describes a woman who has never delivered a potentially live baby
(i.e. gestation over 24 weeks) but she may have had miscarriages or terminations
before 24 weeks gestation. Multiparity describes a woman who has delivered
at least one potentially live baby, i.e. gestation over 24 weeks. It is not
relevant that there was a different partner and the modes of delivery are not
included in this nomenclature.

38
Q

A 25-year-old woman has an early transvaginal ultrasound scan due to some
per vagina brown spotting with a positive pregnancy test. The scan showed
no intrauterine pregnancy. She had two bhCG serum tests taken, the first
on the day of her scan showed a result of 653 and the second 48 hours
later had a result of 623.
What is the most likely diagnosis?

A. Early intrauterine pregnancy

B. Ectopic pregnancy

C. Inevitable miscarriage

D. Missed miscarriage

E. Threatened miscarriage

A

B – Ectopic pregnancy

An ectopic pregnancy means the fertilized ovum has implanted outside the
uterine cavity. The most common place is the fallopian tube but they have
been seen on the ovary, in the cornua of the uterus and in the abdomen.
Investigations for ectopic pregnancies depend on the clinical condition of the
patient. If the patient is well and has no pain or cervical excitation on vaginal
examination, outpatient investigations are considered. This includes serial
bhCG measurements 48 hours apart and the next available ultrasound scan.
In the early stages of a normal pregnancy the bhCG level should double every
48 hours. If the bhCG plateaus an ectopic must be ruled out. If the bhCG is
falling it is likely to represent a miscarriage. A gestational sac should generally
be seen on transvaginal scan if the bhCG level is above 1000 iu (or .6500 iu
for transabdominal scans).
Ectopic pregnancy is the most likely diagnosis in this case. Initially the scan is not
worrying as a gestational sac should not be expected to be visualized with a
bhCG level of 653 iu. The patient was quite rightly discharged and asked to
return 48 hours later for a further bhCG test. She was advised to return
sooner if she had any pain or was concerned by bleeding or feeling unwell.
The second bhCG result has not doubled which would lead us to the diagnosis
of ectopic and further investigation such as a laparoscopy should be considered.
If the bhCG had doubled, a repeat scan in one week could be arranged to ensure
viability of the pregnancy. If the bhCG had rapidly fallen a diagnosis of complete
miscarriage could have been made and the patient would have been followed
up using bhCG.

39
Q

A 42-year-old woman was seen by the general practitioner after she
complained of fatigue, weight loss and more recently a change in bowel
habit. On examination, her abdomen was distended and the doctor elicited
a positive fluid thrill test. She was urgently referred to the bowel surgeons;
however, on CT bilateral ovarian cysts were seen. After referral to the gynaecology
oncologists she had an operation and the histological findings
were of psammoma bodies. Her diagnosis is the most common ovarian
carcinoma.
Which type of ovarian cancer did she have?

A. Clear cell tumour

B. Endometrioid tumour

C. Mucinous tumour

D. Serous tumour

E. Urothelial-like tumour

A

D – Serous tumour

The presentation of ovarian cancer is non-specific and varied, and most present
late. The most common symptoms are pain, which can be caused by bleeding
or torsion, and abdominal swelling. Other symptoms include anorexia, nausea
and vomiting, change in bowel habit, weight loss, abnormal vaginal bleeding,
urinary symptoms, malaise, deep vein thrombosis, and rarely hormonal effects
such as virilization and precocious puberty. Signs include an abdominal/pelvic
mass, ascites, pleural effusion, cervical lymphadenopathy and hepatomegaly.

Spread through the peritoneal cavity leads to intraperitoneal disease which gives
rise to intestinal obstruction and cachexia. More distant spread includes liver
metastases and malignant pleural effusions. Investigations for ovarian cancer
include blood tests for routine measurements and tumour markers, pelvic and
abdominal ultrasound scan, CT and chest X-ray.
Epithelial tumours are the most common of the ovarian tumours and are divided
into serous, mucinous, endometrioid, clear cell and urothelial-like (Brenner)
tumours. Serous tumours comprise approximately half of all ovarian cancers
and are the most common ovarian neoplasm. They occur mainly in women of
reproductive age. The benign form – serous cystadenomas – consists of unilocular
cysts of variable sizes filled with straw-coloured fluid and 20% are bilateral.
The malignant type – serous cystadenocarcinomas – consists of mixed solid
areas with unilocular cysts. Histological findings are psammoma bodies which
are concentric laminated calcified concretions. They are bilateral in 30–50%
cases. Other types include sex cord/stromal tumours including granulosa cell,
thecoma/fibroma tumours and Sertoli/Leydig cell tumours. Germ cell tumours
include dysgerminomas, endodermal sinus or yolk sac tumours, choriocarcinomas
and teratomas (dermoid cyst).

40
Q

A B-lynch brace suture is used in management of this cause of postpartum
haemorrhage.

A. 3rd degree perineal tear

B. 4th degree perineal tear

C. Atonic uterus

D. Cervical tear

E. High vaginal tear

A

C – Atonic uterus

Vaginal blood loss during a normal delivery is expected to be between 200 and
300 mL. A postpartum haemorrhage (PPH) describes blood loss over 500 mL or
less than 500 mL if associated with haemodynamic changes in the mother.
Around 5% of deliveries are reported to have been associated with PPH but
this number is probably higher as blood loss is often underestimated. Lifethreatening
PPH occurs in 1 in 1000 deliveries. Primary PPH occurs within the
first 24 hours and secondary PPH ranges from 24 hours to 6 weeks postpartum.

41
Q

A 59-year-old woman who has had three previous vaginal deliveries
complains of a feeling of ‘something coming down’ at the front of her
vagina and increased urinary frequency. On examination, there is a bulge
at the front of her vagina which is easily visible with a Sims’ speculum. It
is worse when she coughs.
What is the most likely diagnosis?

A. Cystocele

B. Enterocele

C. Procidentia

D. Rectocele

E. Vault prolapse

A

A – Cystocele

Prolapse can be asymptomatic but if symptoms are present they are often nonspecific
general discomfort, dragging, a feeling of a lump or swelling that
worsens as the day progresses and rarely coital problems.
Cystocele describes descent of the bladder through the superior anterior vaginal
wall. This can occur in conjunction with an urethrocele (descent of the first 3 to
4 cm of the vaginal wall which overlies the urethra). Urinary symptoms occur
due to alteration of the urethrovesical angle which can lead to genuine stress
incontinence, urinary frequency or urgency and difficulties in emptying the
bladder which can lead to overflow incontinence. Patients with large cystoceles
are predisposed to urinary tract infections due to incomplete emptying of
bladder.
Uterovaginal prolapse is descent of the pelvic organs into the vagina. Descent of
a single organ rarely occurs due to the close proximity of the urethra, bladder,
uterus, small bowel and rectum. The main predisposing factor allowing descent
is weakening of the pelvic tissues by the trauma of childbirth which is compounded
by menopausal oestrogen deficiency and loss of connective tissue strength.
Other contributing factors to prolapse formation are congenital causes, previous
suprapubic surgery for urinary incontinence and genetic factors. These tend not
to occur in isolation, rather, it is a combination of these factors which are also
aggravated by mechanisms which increase intra-abdominal pressure including
obesity, cough, constipation, heavy lifting, ascites or a large pelvic mass.

42
Q

A 26-year-old female student is seen in the emergency department with
postcoital bleeding which she has experienced for 2 weeks. She has no
other abnormal discharge. She uses the oral contraceptive pill and has a
long-term partner with whom she always uses barrier contraception. Her
last smear was a year ago and was normal. On examination you see an
abnormal area of skin surrounding the os. It is flat with a florid appearance.
There is contact bleeding when a swab is taken.
What is the most likely diagnosis?

A. Cervical cancer

B. Cervical ectropion

C. Cervical polyps

D. Endometrial carcinoma

E. Endometrial polyps

A

B – Cervical ectropion

Cervical ectropion is an entirely benign change in the area of mucosa surrounding
the lower cervical canal. A florid appearance is described (signifying
secretory glandular mucosa). It is often asymptomatic, but may cause postcoital
bleeding or persistent vaginal discharge. There may be no obvious cause but it is
associated with hormonal changes during puberty, pregnancy or the oral
contraceptive pill. Sexually transmitted infections must be ruled out and a
normal smear test must be seen. If there is no reason to suspect anything
more sinister then it can be treated with diathermy or cryocautery.
It would be very rare for a woman this young to present with endometrial carcinoma.
It is generally a disease of older women who present with post menopausal
bleeding. It rarely presents prior to the menopause with abnormal
bleeding, discharge or pyometra (pus in the uterus). Prompt investigations
must be initiated if there is any suspicion of endometrial cancer including hysteroscopy
with endometrial sampling and ultrasound or MRI. Spread is seen
directly through the endometrial cavity and cervix and along the fallopian
tubes to the ovaries and peritoneal cavity.

43
Q

A 30-year-old primigravid woman who is 34 weeks pregnant attends the
antenatal clinic. She has persistent hypertension of 164/112 mmHg and
some protein in her urine on dipstick testing. She has no visual disturbances,
no epigastric pain and complains of mild headaches which are generally
relieved with paracetamol. On examination, her abdomen is soft and
non-tender, she has mild pedal oedema, normal reflexes and one beat of
clonus. Her bloods are all normal. She has asthma for which she uses a
salbutamol inhaler when required.
Which antihypertensive should you use to manage her hypertension?

A. Furosemide

B. Labetalol

C. Magnesium sulphate

D. Nifedipine

E. Ramipril

A

`D – Nifedipine

Pre-eclampsia and eclampsia are pregnancy-specific conditions. There is a
spectrum of pathology which includes gestational hypertension (hypertension
alone), pre-eclampsia (hypertension and proteinuria, with or without oedema
and involvement of other organs), eclampsia (generalized convulsions in the
presence of pre-eclampsia) and HELLP syndrome (Haemolysis, Elevated Liver
enzymes and Low Platelets). Affected women can progress quickly from mild
symptoms to convulsions, although some women look unwell but have no
serious problems. The only cure is delivery of the baby and placenta. Screening
tests of blood pressure and urine dip are done every time the woman sees her
midwife or is seen in antenatal clinic.
Symptoms of pre-eclampsia include headache, blurred vision and epigastric
pain (although most cases are asymptomatic). Clinical signs are high blood
pressure, hyperreflexia, proteinuria, vomiting, hepatic tenderness, oliguria and
spontaneous bleeding. Classification of pre-eclampsia is as follows: mild
(blood pressure (BP) .140/90 mmHg with proteinuria of .0.3 g in 24 hours),
moderate (diastolic BP 100–110 mmHg with proteinuria) and severe
(BP .160 mmHg systolic or .110 mmHg diastolic, with proteinuria). The BP
readings must be taken with the patient resting, with at least two readings 6
hours apart. Indicators of severe disease are cerebral or visual disturbances,
abdominal pain, fetal growth restriction, oliguria (urine output ,500 mL in 24
hours), impaired liver function tests and thrombocytopenia.
Pre-eclampsia has no definite pathophysiology but it appears to be caused by a
placental factor which leads to endothelial dysfunction. Maternal risk factors
for pre-eclampsia include first pregnancy, obesity, family history, extremes of
maternal age, pre-existing hypertension, congenital thrombophilia, systemic
lupus erythematosus, antiphospholipid disease, renal disease, diabetes, multiple
pregnancy, hydatidiform mole and hydrops fetalis.
Blood pressure control
Women with BP systolic .160 or diastolic .110 should be started on antihypertensives
(although the threshold for treatment is lower in women with severe
symptoms). Antihypertensives do not ‘cure’ the disease but it may avoid
extreme prematurity by limiting the risk of vascular damage due to uncontrolled
hypertension. Antihypertensive medication may need to be continued for 3
months postpartum.

44
Q

A 31-year-old nulliparous woman is requesting the combined oral contraceptive
pill. She says she would like to consider trying to conceive in 2
years time. On further questioning she reveals she smokes 15 cigarettes
per day, she occasionally suffers from migraines with typical focal aura
and her mother had a pulmonary embolism when aged 50 associated
with abdominal surgery. She has a body mass index of 26. There is no
other relevant history.
Can you safely prescribe her the pill and if not, why not?

A. No – family history of thromboembolic disease

B. No – intends on having children in future

C. No – migraines

D. No – smoker

E. Yes

A

C – No – migraines

Migraines with typical focal aura are a contraindication to the combined oral
contraceptive pill (COCP).
COCP is widely used as first-line contraception and over 99% effective if taken
appropriately. They prevent ovulation, thicken cervical mucus to prevent
sperm reaching an egg and thin the lining of the womb to prevent ovum implantation
by the action of two hormones, oestrogen and progesterone. The advantages
include a reduction in period pains, menstrual bleeding and premenstrual
symptoms. They protect against cancer of the ovary, uterus and colon, as well as
some pelvic infections. Fertility quickly returns to normal when the medication is
discontinued so this would not be a reason not to prescribe the COCP for this
lady. The disadvantages include temporary minor side-effects such as headaches,
mood changes and breast tenderness, and there is a slightly increased risk of
venous thrombosis, breast cancer and cervical cancer. COCPs are less effective
with some drugs, vomiting or severe long-lasting diarrhoea or if pills are
missed. In such instances, additional contraception needs to be used.
There are many contraindications to combined oral contraceptives – thromboembolism
(below), heart/arterial disease (below), migraine (below), transient
ischaemic attack (TIA), liver disease, infective hepatitis, systemic lupus erythematosus,
porphyria, liver adenoma, gallstones, post-evacuation of hydatidiform
mole, haemolytic uraemic syndrome, history during pregnancy of pruritis, cholestatic
jaundice, chorea or pemphigoid gestations, breast or genital carcinoma,
undiagnosed vaginal bleeding, breastfeeding and pregnancy.

45
Q

A 27-year-old para 1þ0 woman attends the labour ward with regular
painful contractions at 31 weeks gestation. On abdominal examination, a
cephalic presentation is felt with the head 3/5 palpable. The contractions
are palpable and regular every 3 minutes lasting 30 seconds.
What drug would you give to reduce the risk of respiratory distress
syndrome if the baby is born prematurely?

A. Amoxicillin

B. Betamethasone

C. Nifedipine

D. Salbutamol

E. Theophylline

A

B – Betamethasone

Premature labour is labour before 37 weeks. It occurs in 5–10% pregnancies
with a rising incidence but 5% of women that have threatened preterm
labour actually go on to deliver at term. Survival rates are improving with
advances in neonatal medicine but many believe the limit of viability is being
reached at 24 weeks gestation. Onset of labour is suggested by the presence
of painful regular contractions associated with cervical change although there
may only be some lower abdominal pain or silent dilation of the cervix.
The pathophysiology of preterm labour is not fully understood but there are a
number of risk factors, the greatest of which is previous preterm labour.
Maternal factors include extremes of maternal age, substance abuse,
smoking, infection including urinary tract infection, genital tract infection or
chorioamnionitis, low socioeconomic status, lack of prenatal care, intense physical
exertion or stress, uterine distension such as polyhydramnios or multiple
pregnancies, pre-eclampsia, antepartum haemorrhage, placental abruption
and uterine abnormalities such as cervical incompetence or bicornuate uterus
or previous cervical surgery. Fetal factors include pre-labour rupture of membranes,
multiple pregnancy, intrauterine growth restriction and congenital
abnormalities.

46
Q

A 24-year-old woman, who has been with her current partner for 5 years,
has been trying to conceive for 2 years. They are now being investigated
for subfertility. The woman is being investigated for tubal patency.
What is the gold standard test for tubal patency?

A. Laparoscopy and dye

B. Hysterosalpingo-contrast sonography

C. Hysterosalpingogram

D. Hysteroscopy

E. Salpingoscopy

A

A – Laparoscopy and dye

Diagnostic laparoscopy and dye is the gold standard test for tubal patency. A
laparoscopy is performed, under general anaesthetic, to give a direct view of
pelvic organs to assess abnormalities, damage or significant adhesions. During
the operation, methylene blue dye is inserted into the uterus using a syringe
via the cervix. If blue dye is seen coming from the fimbrial ends of the fallopian
tubes they are deemed patent. Hysteroscopy is often done at the same time.
Hysterosalpingogram involves passing radio-opaque fluid into uterine cavity via
the cervix to ascertain if it leaks from the ends of the fallopian tube via X-ray.
If it is normal the result can be trusted 97% of the time. If it is abnormal, it is
only correct in 34% of cases, and a laparoscopy and dye test would be
needed to confirm the test. The advantages of this overlap and dye are that
this avoids the risk of surgery and anaesthetic and it can be done as an outpatient
procedure. It is generally done in women with no risk factors for pelvic
disease or those with high anaesthetic risk.
Salpingoscopy involves inserting a fine telescope (salpingoscope) into the ampullary
portion of the fallopian tube at laparoscopy to investigate the presence of
fine intratubal adhesions. Falloposcopy involves insertion of a fibreoptic instrument
(falloposcope) into the fallopian tube via the uterine cavity as an outpatient
procedure. This is very new and is generally only used as a research tool as it
is expensive and delicate. Hysterosalpingo-contrast sonography involves inserting a galactose-containing ultrasound contrast medium into the uterine cavity.
A standard ultrasound probe is then used to outline abnormalities such as submucosal
fibroids, endometrial polyps and the patency of the fallopian tubes. It
has a similar accuracy to the hysterosalpingogram.

47
Q

A 32-year-old pregnant woman attends the emergency department with a
history of bleeding per vagina. She says the bleeding is lighter than her
normal period and has lasted for 2 days. She is 9 weeks gestation and this
is her first pregnancy. She says she knows she is rhesus D negative as she
gives blood regularly.
What needs to be done today with regards to anti-D prophylaxis?

A. Give antenatal anti-D prophylaxis 250 iu

B. Give antenatal anti-D prophylaxis 500 iu

C. Give postnatal anti-D

D. Give routine antenatal anti-D prophylaxis at 28 weeks

E. No action needed at present

A

E – No action needed at present

It sounds like this woman has had a threatened miscarriage. The guidelines state
that only those women with per vagina bleeding after 12 weeks require anti-D.
As long as the bleeding is not heavy this woman could be sent home after arranging
a pelvic scan to assess the bleeding and viability of the pregnancy.
The RCOG guidelines provide advice on who should be given anti-D prophylaxis
in the early stages of pregnancy. Threatened miscarriage: Anti-D prophylaxis
should be given to all RhD –ve women after 12 weeks in the event of a threatened
miscarriage but not given before 12 weeks as is seen in this case. Spontaneous
miscarriage: A similar distinction is given for spontaneous miscarriage
with those over 12 weeks requiring anti-D prophylaxis but not for those
under 12 weeks gestation unless uterine evacuation is required. Therapeutic
termination of pregnancy: Anti-D prophylaxis is needed in all non-sensitized
women having medical or surgical termination of pregnancy regardless of
gestational age. Ectopic pregnancy: Anti-D prophylaxis should be given to all
non-sensitised women who have ectopic pregnancy.

48
Q

A 29-year-old woman who is 38 weeks þ 3 days attends the antenatal clinic
after being referred by the midwife for measuring smaller than her dates.
The baby is on the 25th centile for all measurements and there is a
normal liquor volume. The placental site is fundal.
Which emergency does this make her more at risk of when she is in labour?

A. Cord prolapse

B. Fetal distress

C. Stillbirth

D. Uterine inversion

E. Uterine rupture
Practice

A

D – Uterine inversion

Uterine inversion is rare and describes the passage of the uterine fundus through
the cervix into the vagina. The resultant stretch on the round ligament can cause
profound shock due to vagal stimulation. The shock seen is out of proportion to
blood loss and this increases suspicion of uterine inversion rather than haemorrhage.
A fundal placenta means there is an increased risk of uterine inversion, so it is
particularly important not to attempt to remove the placenta before signs of placental
separation have been seen (cord lengthening and a gush of dark blood).
It is also important to only use controlled cord traction with counter-pressure on
the uterus over the suprapubic region. Other risk factors for inversion are uterine
atony and previous uterine inversion.
Presentation includes shock (primarily vasovagal), which is out of proportion to
the amount of bleeding and pain. The uterine fundus will not be palpable
abdominally – rather it will be seen or felt as a bluish-grey mass in the vagina.
The placenta remains attached in 50% of cases and can be morbidly adherent
so further attempts to remove it should not be attempted.

49
Q

Which of the following is the most common cause of primary postpartum
haemorrhage?

A. Atonic uterus

B. Disseminated intravascular coagulation

C. Infection

D. Perineal trauma

E. Retained placental fragments

A

A – Atonic uterus

Ninety percent of primary postpartum haemorrhages (PPHs) are caused by an
atonic uterus. Other causes include cervical tear, high vaginal tear, perineal
trauma, retained placenta or placental fragments, clotting disorders, uterine
inversion and uterine rupture. Risk factors include antepartum haemorrhage,
previous history of PPH, over-enlarged uterus due to multiple pregnancy, polyhydramnios
or macrosomic fetus, uterine fibroids, placenta praevia, prolonged
labour, grand multiparity, chorioamnionitis and bleeding diathesis.
It is easy to remember the causes of PPH as the four Ts:
† Tone
† Tissue
† Trauma
† Thrombosis – i.e. clotting disorders
Uterine atony occurs due to inadequate compression of the intramyometrial
blood vessels by the uterine muscles leading to continued bleeding. It is more
common if there is a retained placenta or placental tissue as the physical presence
of this prevents contraction of the uterus occurring.
Initial management of an atonic uterus includes emptying the bladder and
‘rubbing up uterine contractions’ while administering oxytocics to contract
the uterus. To ‘rub up a contraction’ the uterus is massaged through the
abdominal wall. A bolus of intravenous ergometrine is given followed by infusion
of oxytocin. If bleeding persists, bimanual compression can be employed
which involves inserting a fist into the vagina and pressing the uterus onto it
to compress it. For further persistent bleeding, carboprost (15-methylprostaglandin/
Haemobate) can be given intramuscularly into the thigh or gluteal
muscle. If the bleeding is not controlled surgical options must be considered.

50
Q

Which of the following is the by-product of female gametogenesis?

A. Mature oocyte

B. Oogonia

C. Polar body

D. Primary oocyte

E. Secondary oocyte

A

C – Polar body

Oogenesis begins in females during intrauterine life and is a slow process with
many stages of division being arrested for long periods of time. It is more complicated
than the male pathway, although the same principles are
followed. Primordial germ cells are present in the wall of the yolk sac at the
end of the third week of embryological development. They migrate to the developing
gonads by amoeboid movement where they arrive by the beginning of
the 5th week. They then divide by mitosis and differentiate into oogonia.
Oogonia differentiate into primary oocytes and become surrounded by follicular
(epithelial) cells. These are collectively named primordial follicles. Primary
oocytes contain 46 double-structured chromosomes and enter prophase of
meiosis I. Division is arrested at the dictyotene phase of meiosis I, and the first
meiotic division is completed only with the pre-ovulatory leutenizing
hormone/follicle-stimulating hormone (LH/FSH) surge to give a secondary
oocyte and a polar body. The polar body is merely a useless by-product which
subsequently degenerates.
Secondary oocyte and polar bodies both contain 23 double-structured chromosomes.
Meiosis I of the secondary oocyte is completed with the pre-ovulatory
LH/FSH surge, but further division of the secondary oocyte only occurs if fertilization
occurs. With fertilization, the secondary oocyte divides by meiosis II to
give a mature oocyte and a further polar body. The mature oocytes and its
polar body both contain 23 single chromosomes. The initial polar body also
undergoes meiosis II to give a further two polar bodies, both containing 23
single chromosomes.
Primary oocytes therefore eventually give rise to four daughter cells, all of which
contain 22 þ X chromosomes. However, only one of these is a mature oocyte,
the remaining three daughter cells being polar bodies.